Can this approximate closed form of Apery's constant $zeta(3)$ be improved?












13












$begingroup$


I know that an approximate closed form is not really a solution. However, I would like to present a method that gives a closed form of $zeta(3)$ that is accurate to the 5th decimal, hoping that it may help to find the exact expression



Consider a real function:
$g(x)=-frac{1}{2!}+frac{x^3}{5!}-frac{x^6}{8!}+frac{x^{9}}{11!}-...$



This function $g(x)$ has infinite positive roots. Let's denote these roots by $r_i$. Because $g(x)$ is a function of $x^3$ every root $r_i$ should be a triple root. We can show that:



$g(x)=-frac{1}{2}prod_{i=1}^{infty}{(1-frac{x^3}{r_i^3})}$



On the other hand, roots of $g(x)=0$ are given by:
$2sin(frac{pi}{6}-frac{sqrt{3}}{2}x)=e^{-frac{3x}{2}}$. As $x$ increases, the exponential part assimptotes to zero and roots of the function become closer to the roots of a sine wave $2sin(frac{pi}{6}-frac{sqrt{3}}{2}x)=0$. The roots of sine equation are easy to show:$mu_i=frac{(6i+1)pi}{3sqrt{3}}, i=1,2,3,... $



Now, assuming that these two sets of roots ($r_i$ and $mu_i$) are close to eachother, we can write:
$g(x)cong-frac{1}{2}prod_{i=1}^{infty}(1-frac{x^3}{mu_i^3})$
or
$g(x)cong-frac{1}{2}prod_{i=1}^{infty}(1-frac{(3sqrt{3})^3x^3}{(6i+1)^3pi^3})$



The coefficient for the $x^3$ must be equal to that of the infinite series of $g(x)$.
$sum_{i=1}^{infty}frac{(3sqrt{3})^3x^3}{(6i+1)^3pi^3}congfrac{2x^3}{5!}$



and $sum_{i=1}^{infty}frac{1}{(6i+1)^3}congfrac{2pi^3}{5!(3sqrt{3})^3}$ and we know that $sum_{i=1}^{infty}frac{1}{(6i+1)^3}=frac{(91zeta(3)+2sqrt{3}pi^3)}{216}-1$ combining these two equations gives us an approximate closed form of $zeta(3)$:



$zeta(3)congfrac{4(7290-67sqrt{3}pi^3)}{12285}$



which is accurate to the fifth decimal. This method is analogous to Euler's derivation of $zeta(2)$ except that the roots of this function $g(x)$ are slightly nonlinear. Is it possible to reduce the error in the approximation of the roots $r_i$ by $mu_i$?










share|cite|improve this question











$endgroup$








  • 3




    $begingroup$
    A really interesting Eulerian-like method.
    $endgroup$
    – Jack D'Aurizio
    Oct 1 '15 at 15:15






  • 1




    $begingroup$
    I allow myself to think the young Majid would love to prove the Riemann Hypothesis. Who knows? Best wishes for him.
    $endgroup$
    – Piquito
    Oct 1 '15 at 20:01










  • $begingroup$
    @Ataulfo Thanks very much for your kind words.
    $endgroup$
    – Majid Fekri
    Oct 2 '15 at 18:38










  • $begingroup$
    There is one minor issue with your work that I noticed. $1-frac{x^3}{r^3}$ has three distinct complex roots, since every number except zero has three distinct complex third roots. Thus, $romega$ and $romega^2$ are also roots with $omega$ as the first complex third root of unity. I am not sure that hurts anything, though, with the result.
    $endgroup$
    – Terra Hyde
    Nov 25 '16 at 14:57








  • 1




    $begingroup$
    @TerraHyde: The roots come in form of complex triplets but if you multiply every three of them together they always form ${1-frac{x^3}{r_i^3}}$ where ${r_i}$ is real.
    $endgroup$
    – Majid Fekri
    Nov 29 '16 at 21:46


















13












$begingroup$


I know that an approximate closed form is not really a solution. However, I would like to present a method that gives a closed form of $zeta(3)$ that is accurate to the 5th decimal, hoping that it may help to find the exact expression



Consider a real function:
$g(x)=-frac{1}{2!}+frac{x^3}{5!}-frac{x^6}{8!}+frac{x^{9}}{11!}-...$



This function $g(x)$ has infinite positive roots. Let's denote these roots by $r_i$. Because $g(x)$ is a function of $x^3$ every root $r_i$ should be a triple root. We can show that:



$g(x)=-frac{1}{2}prod_{i=1}^{infty}{(1-frac{x^3}{r_i^3})}$



On the other hand, roots of $g(x)=0$ are given by:
$2sin(frac{pi}{6}-frac{sqrt{3}}{2}x)=e^{-frac{3x}{2}}$. As $x$ increases, the exponential part assimptotes to zero and roots of the function become closer to the roots of a sine wave $2sin(frac{pi}{6}-frac{sqrt{3}}{2}x)=0$. The roots of sine equation are easy to show:$mu_i=frac{(6i+1)pi}{3sqrt{3}}, i=1,2,3,... $



Now, assuming that these two sets of roots ($r_i$ and $mu_i$) are close to eachother, we can write:
$g(x)cong-frac{1}{2}prod_{i=1}^{infty}(1-frac{x^3}{mu_i^3})$
or
$g(x)cong-frac{1}{2}prod_{i=1}^{infty}(1-frac{(3sqrt{3})^3x^3}{(6i+1)^3pi^3})$



The coefficient for the $x^3$ must be equal to that of the infinite series of $g(x)$.
$sum_{i=1}^{infty}frac{(3sqrt{3})^3x^3}{(6i+1)^3pi^3}congfrac{2x^3}{5!}$



and $sum_{i=1}^{infty}frac{1}{(6i+1)^3}congfrac{2pi^3}{5!(3sqrt{3})^3}$ and we know that $sum_{i=1}^{infty}frac{1}{(6i+1)^3}=frac{(91zeta(3)+2sqrt{3}pi^3)}{216}-1$ combining these two equations gives us an approximate closed form of $zeta(3)$:



$zeta(3)congfrac{4(7290-67sqrt{3}pi^3)}{12285}$



which is accurate to the fifth decimal. This method is analogous to Euler's derivation of $zeta(2)$ except that the roots of this function $g(x)$ are slightly nonlinear. Is it possible to reduce the error in the approximation of the roots $r_i$ by $mu_i$?










share|cite|improve this question











$endgroup$








  • 3




    $begingroup$
    A really interesting Eulerian-like method.
    $endgroup$
    – Jack D'Aurizio
    Oct 1 '15 at 15:15






  • 1




    $begingroup$
    I allow myself to think the young Majid would love to prove the Riemann Hypothesis. Who knows? Best wishes for him.
    $endgroup$
    – Piquito
    Oct 1 '15 at 20:01










  • $begingroup$
    @Ataulfo Thanks very much for your kind words.
    $endgroup$
    – Majid Fekri
    Oct 2 '15 at 18:38










  • $begingroup$
    There is one minor issue with your work that I noticed. $1-frac{x^3}{r^3}$ has three distinct complex roots, since every number except zero has three distinct complex third roots. Thus, $romega$ and $romega^2$ are also roots with $omega$ as the first complex third root of unity. I am not sure that hurts anything, though, with the result.
    $endgroup$
    – Terra Hyde
    Nov 25 '16 at 14:57








  • 1




    $begingroup$
    @TerraHyde: The roots come in form of complex triplets but if you multiply every three of them together they always form ${1-frac{x^3}{r_i^3}}$ where ${r_i}$ is real.
    $endgroup$
    – Majid Fekri
    Nov 29 '16 at 21:46
















13












13








13


10



$begingroup$


I know that an approximate closed form is not really a solution. However, I would like to present a method that gives a closed form of $zeta(3)$ that is accurate to the 5th decimal, hoping that it may help to find the exact expression



Consider a real function:
$g(x)=-frac{1}{2!}+frac{x^3}{5!}-frac{x^6}{8!}+frac{x^{9}}{11!}-...$



This function $g(x)$ has infinite positive roots. Let's denote these roots by $r_i$. Because $g(x)$ is a function of $x^3$ every root $r_i$ should be a triple root. We can show that:



$g(x)=-frac{1}{2}prod_{i=1}^{infty}{(1-frac{x^3}{r_i^3})}$



On the other hand, roots of $g(x)=0$ are given by:
$2sin(frac{pi}{6}-frac{sqrt{3}}{2}x)=e^{-frac{3x}{2}}$. As $x$ increases, the exponential part assimptotes to zero and roots of the function become closer to the roots of a sine wave $2sin(frac{pi}{6}-frac{sqrt{3}}{2}x)=0$. The roots of sine equation are easy to show:$mu_i=frac{(6i+1)pi}{3sqrt{3}}, i=1,2,3,... $



Now, assuming that these two sets of roots ($r_i$ and $mu_i$) are close to eachother, we can write:
$g(x)cong-frac{1}{2}prod_{i=1}^{infty}(1-frac{x^3}{mu_i^3})$
or
$g(x)cong-frac{1}{2}prod_{i=1}^{infty}(1-frac{(3sqrt{3})^3x^3}{(6i+1)^3pi^3})$



The coefficient for the $x^3$ must be equal to that of the infinite series of $g(x)$.
$sum_{i=1}^{infty}frac{(3sqrt{3})^3x^3}{(6i+1)^3pi^3}congfrac{2x^3}{5!}$



and $sum_{i=1}^{infty}frac{1}{(6i+1)^3}congfrac{2pi^3}{5!(3sqrt{3})^3}$ and we know that $sum_{i=1}^{infty}frac{1}{(6i+1)^3}=frac{(91zeta(3)+2sqrt{3}pi^3)}{216}-1$ combining these two equations gives us an approximate closed form of $zeta(3)$:



$zeta(3)congfrac{4(7290-67sqrt{3}pi^3)}{12285}$



which is accurate to the fifth decimal. This method is analogous to Euler's derivation of $zeta(2)$ except that the roots of this function $g(x)$ are slightly nonlinear. Is it possible to reduce the error in the approximation of the roots $r_i$ by $mu_i$?










share|cite|improve this question











$endgroup$




I know that an approximate closed form is not really a solution. However, I would like to present a method that gives a closed form of $zeta(3)$ that is accurate to the 5th decimal, hoping that it may help to find the exact expression



Consider a real function:
$g(x)=-frac{1}{2!}+frac{x^3}{5!}-frac{x^6}{8!}+frac{x^{9}}{11!}-...$



This function $g(x)$ has infinite positive roots. Let's denote these roots by $r_i$. Because $g(x)$ is a function of $x^3$ every root $r_i$ should be a triple root. We can show that:



$g(x)=-frac{1}{2}prod_{i=1}^{infty}{(1-frac{x^3}{r_i^3})}$



On the other hand, roots of $g(x)=0$ are given by:
$2sin(frac{pi}{6}-frac{sqrt{3}}{2}x)=e^{-frac{3x}{2}}$. As $x$ increases, the exponential part assimptotes to zero and roots of the function become closer to the roots of a sine wave $2sin(frac{pi}{6}-frac{sqrt{3}}{2}x)=0$. The roots of sine equation are easy to show:$mu_i=frac{(6i+1)pi}{3sqrt{3}}, i=1,2,3,... $



Now, assuming that these two sets of roots ($r_i$ and $mu_i$) are close to eachother, we can write:
$g(x)cong-frac{1}{2}prod_{i=1}^{infty}(1-frac{x^3}{mu_i^3})$
or
$g(x)cong-frac{1}{2}prod_{i=1}^{infty}(1-frac{(3sqrt{3})^3x^3}{(6i+1)^3pi^3})$



The coefficient for the $x^3$ must be equal to that of the infinite series of $g(x)$.
$sum_{i=1}^{infty}frac{(3sqrt{3})^3x^3}{(6i+1)^3pi^3}congfrac{2x^3}{5!}$



and $sum_{i=1}^{infty}frac{1}{(6i+1)^3}congfrac{2pi^3}{5!(3sqrt{3})^3}$ and we know that $sum_{i=1}^{infty}frac{1}{(6i+1)^3}=frac{(91zeta(3)+2sqrt{3}pi^3)}{216}-1$ combining these two equations gives us an approximate closed form of $zeta(3)$:



$zeta(3)congfrac{4(7290-67sqrt{3}pi^3)}{12285}$



which is accurate to the fifth decimal. This method is analogous to Euler's derivation of $zeta(2)$ except that the roots of this function $g(x)$ are slightly nonlinear. Is it possible to reduce the error in the approximation of the roots $r_i$ by $mu_i$?







riemann-zeta






share|cite|improve this question















share|cite|improve this question













share|cite|improve this question




share|cite|improve this question








edited Nov 29 '16 at 22:02







Majid Fekri

















asked Oct 1 '15 at 15:10









Majid FekriMajid Fekri

1017




1017








  • 3




    $begingroup$
    A really interesting Eulerian-like method.
    $endgroup$
    – Jack D'Aurizio
    Oct 1 '15 at 15:15






  • 1




    $begingroup$
    I allow myself to think the young Majid would love to prove the Riemann Hypothesis. Who knows? Best wishes for him.
    $endgroup$
    – Piquito
    Oct 1 '15 at 20:01










  • $begingroup$
    @Ataulfo Thanks very much for your kind words.
    $endgroup$
    – Majid Fekri
    Oct 2 '15 at 18:38










  • $begingroup$
    There is one minor issue with your work that I noticed. $1-frac{x^3}{r^3}$ has three distinct complex roots, since every number except zero has three distinct complex third roots. Thus, $romega$ and $romega^2$ are also roots with $omega$ as the first complex third root of unity. I am not sure that hurts anything, though, with the result.
    $endgroup$
    – Terra Hyde
    Nov 25 '16 at 14:57








  • 1




    $begingroup$
    @TerraHyde: The roots come in form of complex triplets but if you multiply every three of them together they always form ${1-frac{x^3}{r_i^3}}$ where ${r_i}$ is real.
    $endgroup$
    – Majid Fekri
    Nov 29 '16 at 21:46
















  • 3




    $begingroup$
    A really interesting Eulerian-like method.
    $endgroup$
    – Jack D'Aurizio
    Oct 1 '15 at 15:15






  • 1




    $begingroup$
    I allow myself to think the young Majid would love to prove the Riemann Hypothesis. Who knows? Best wishes for him.
    $endgroup$
    – Piquito
    Oct 1 '15 at 20:01










  • $begingroup$
    @Ataulfo Thanks very much for your kind words.
    $endgroup$
    – Majid Fekri
    Oct 2 '15 at 18:38










  • $begingroup$
    There is one minor issue with your work that I noticed. $1-frac{x^3}{r^3}$ has three distinct complex roots, since every number except zero has three distinct complex third roots. Thus, $romega$ and $romega^2$ are also roots with $omega$ as the first complex third root of unity. I am not sure that hurts anything, though, with the result.
    $endgroup$
    – Terra Hyde
    Nov 25 '16 at 14:57








  • 1




    $begingroup$
    @TerraHyde: The roots come in form of complex triplets but if you multiply every three of them together they always form ${1-frac{x^3}{r_i^3}}$ where ${r_i}$ is real.
    $endgroup$
    – Majid Fekri
    Nov 29 '16 at 21:46










3




3




$begingroup$
A really interesting Eulerian-like method.
$endgroup$
– Jack D'Aurizio
Oct 1 '15 at 15:15




$begingroup$
A really interesting Eulerian-like method.
$endgroup$
– Jack D'Aurizio
Oct 1 '15 at 15:15




1




1




$begingroup$
I allow myself to think the young Majid would love to prove the Riemann Hypothesis. Who knows? Best wishes for him.
$endgroup$
– Piquito
Oct 1 '15 at 20:01




$begingroup$
I allow myself to think the young Majid would love to prove the Riemann Hypothesis. Who knows? Best wishes for him.
$endgroup$
– Piquito
Oct 1 '15 at 20:01












$begingroup$
@Ataulfo Thanks very much for your kind words.
$endgroup$
– Majid Fekri
Oct 2 '15 at 18:38




$begingroup$
@Ataulfo Thanks very much for your kind words.
$endgroup$
– Majid Fekri
Oct 2 '15 at 18:38












$begingroup$
There is one minor issue with your work that I noticed. $1-frac{x^3}{r^3}$ has three distinct complex roots, since every number except zero has three distinct complex third roots. Thus, $romega$ and $romega^2$ are also roots with $omega$ as the first complex third root of unity. I am not sure that hurts anything, though, with the result.
$endgroup$
– Terra Hyde
Nov 25 '16 at 14:57






$begingroup$
There is one minor issue with your work that I noticed. $1-frac{x^3}{r^3}$ has three distinct complex roots, since every number except zero has three distinct complex third roots. Thus, $romega$ and $romega^2$ are also roots with $omega$ as the first complex third root of unity. I am not sure that hurts anything, though, with the result.
$endgroup$
– Terra Hyde
Nov 25 '16 at 14:57






1




1




$begingroup$
@TerraHyde: The roots come in form of complex triplets but if you multiply every three of them together they always form ${1-frac{x^3}{r_i^3}}$ where ${r_i}$ is real.
$endgroup$
– Majid Fekri
Nov 29 '16 at 21:46






$begingroup$
@TerraHyde: The roots come in form of complex triplets but if you multiply every three of them together they always form ${1-frac{x^3}{r_i^3}}$ where ${r_i}$ is real.
$endgroup$
– Majid Fekri
Nov 29 '16 at 21:46












1 Answer
1






active

oldest

votes


















2












$begingroup$

First of all, thank you for the highly interesting question, it provides an excellent exercise!





Unfortunately it is not possible to improve the approximation by your method alone. Of course, the error in your approximation is due to taking the roots of $2sin(frac{pi}{6}-frac{sqrt{3}}{2}x)=e^{-frac{3x}{2}}$ when $xtoinfty$ and thus using the asymptotic $2sin(frac{pi}{6}-frac{sqrt{3}}{2}x)=0$ instead. In order to alter this error, we examine your function $g(x)$:



$$
g(x)=sum_{n=0}^{infty} frac{(-1)^{n+1}x^{3n}}{(3n+2)!}=frac{e^{-x}left(2e^{3x/2}sin(frac{1}{6}(pi-3sqrt3 x))-1right)}{3x^2},
$$



and note that in order to change the properties of the sine wave on the RHS we need to alter the factorial on the LHS. Let $k$ now denote a positive integer $geq0$ and examine what happens when we change $(3n+2)!$ to some $(3n+k)!$. In particular, we examine the three cases $kequiv0 (textrm{mod} 3)$, $kequiv1 (textrm{mod} 3)$ and $kequiv2 (textrm{mod} 3)$.





Case 1. $k=3j$, $jinmathbb{Z}$.



Let's examine $k=0$ first and generalize later.



$$
g'(x)=sum_{n=0}^{infty} frac{(-1)^{n+1}x^{3n}}{(3n)!}=-frac13 e^{-x} left(2e^{3x/2}cosleft(frac{sqrt{3} x}{2}right)+1right),
$$



and by similar methods we get $zeta(3)cong frac87left(frac{pi^{3}}{6(3)^{3/2}}right)=1.136602043ldots$ which is not a very good approximation. Having $j>0$ yields similar results that include the cosine function, but with some leading $exp$ terms. For instance, we have for $j=3$:



$$
sum_{n=0}^{infty} frac{(-1)^{n+1}x^{3n}}{(3n+9)!}=frac{e^{-x}left(-e^{x}x^6+120e^xx^3-720e^x+480e^{3x/2}cos(frac{sqrt{3} x}{2})+240right)}{720x^9}
,$$



and we can see that when $xtoinfty$ we are making an even bigger error in our approximation by assuming that its roots are those that satisfy the equation



$$
480cosleft(frac{sqrt{3} x}{2}right)=0.
$$



In fact, as $j$ increases so does our error.





Case 2. $k=3j+1$, $jinmathbb{Z}$.



In this case we get similar expressions as your original sine wave, but with some differences in sign (pun not intended). For $j=0$ we have



$$
g'(x)=sum_{n=0}^{infty} frac{(-1)^{n+1}x^{3n}}{(3n+1)!}=-frac{e^{-x}left(2e^{3x/2}sin(frac{1}{6}(pi+3sqrt3 x))-1right)}{3x^2},
$$



which has the same roots as your original function $g(x)$ and the same error in approximating $zeta(3)$. As in Case 1, increasing the value of $j$ leads to even more leading $exp$ terms and thus increases the error in our approximation.





Case 3. $k=3j+2$, $jinmathbb{Z}$.



The case $j=0$ corresponds to your original function $g(x)$. As with cases 1 and 2, increasing $j$ increases the amount of leading $exp$ terms and thus the error in the approximation due to taking the roots of the asymptotic value when $xtoinfty$.





It remains to be seen whether it is possible to sensibly generalize the factorial to $(in+j)!$, $i,jinmathbb{Z}, i>0$ in order to get other approximations for other odd zeta values, perhaps several of these cases might yield good approximations. Euler himself tried to generalize his proof of the Basel problem to compute values for $sum_n n^{-(2k+1)}$, $k>0$ but found no way to achieve this. Since then, considerable brain power has been expended to do just that, without success. I believe that is a good indicator that this path might not lead to a nice closed-form expression for $zeta(3)$ in terms of $pi^3$.





EDIT 11.12.2018



Taking the function



$$
sum_{n=0}^{infty} frac{(-1)^{n+1}x^{5n}}{(3n+2)!}=frac{e^{-x^{5/3}}left(2e^{3x^{5/3}/2}sin(frac{1}{6}(pi-3sqrt3 x^{5/3}))-1right)}{3x^{10/3}}
$$



yields the same approximation as $g(x)$, i.e., with $x^{3n}$ instead of $x^{5n}$ in the sum. This provides further indication that this approach is unlinkely to yield any better results on $zeta(3)$.






share|cite|improve this answer











$endgroup$













    Your Answer





    StackExchange.ifUsing("editor", function () {
    return StackExchange.using("mathjaxEditing", function () {
    StackExchange.MarkdownEditor.creationCallbacks.add(function (editor, postfix) {
    StackExchange.mathjaxEditing.prepareWmdForMathJax(editor, postfix, [["$", "$"], ["\\(","\\)"]]);
    });
    });
    }, "mathjax-editing");

    StackExchange.ready(function() {
    var channelOptions = {
    tags: "".split(" "),
    id: "69"
    };
    initTagRenderer("".split(" "), "".split(" "), channelOptions);

    StackExchange.using("externalEditor", function() {
    // Have to fire editor after snippets, if snippets enabled
    if (StackExchange.settings.snippets.snippetsEnabled) {
    StackExchange.using("snippets", function() {
    createEditor();
    });
    }
    else {
    createEditor();
    }
    });

    function createEditor() {
    StackExchange.prepareEditor({
    heartbeatType: 'answer',
    autoActivateHeartbeat: false,
    convertImagesToLinks: true,
    noModals: true,
    showLowRepImageUploadWarning: true,
    reputationToPostImages: 10,
    bindNavPrevention: true,
    postfix: "",
    imageUploader: {
    brandingHtml: "Powered by u003ca class="icon-imgur-white" href="https://imgur.com/"u003eu003c/au003e",
    contentPolicyHtml: "User contributions licensed under u003ca href="https://creativecommons.org/licenses/by-sa/3.0/"u003ecc by-sa 3.0 with attribution requiredu003c/au003e u003ca href="https://stackoverflow.com/legal/content-policy"u003e(content policy)u003c/au003e",
    allowUrls: true
    },
    noCode: true, onDemand: true,
    discardSelector: ".discard-answer"
    ,immediatelyShowMarkdownHelp:true
    });


    }
    });














    draft saved

    draft discarded


















    StackExchange.ready(
    function () {
    StackExchange.openid.initPostLogin('.new-post-login', 'https%3a%2f%2fmath.stackexchange.com%2fquestions%2f1459709%2fcan-this-approximate-closed-form-of-aperys-constant-zeta3-be-improved%23new-answer', 'question_page');
    }
    );

    Post as a guest















    Required, but never shown

























    1 Answer
    1






    active

    oldest

    votes








    1 Answer
    1






    active

    oldest

    votes









    active

    oldest

    votes






    active

    oldest

    votes









    2












    $begingroup$

    First of all, thank you for the highly interesting question, it provides an excellent exercise!





    Unfortunately it is not possible to improve the approximation by your method alone. Of course, the error in your approximation is due to taking the roots of $2sin(frac{pi}{6}-frac{sqrt{3}}{2}x)=e^{-frac{3x}{2}}$ when $xtoinfty$ and thus using the asymptotic $2sin(frac{pi}{6}-frac{sqrt{3}}{2}x)=0$ instead. In order to alter this error, we examine your function $g(x)$:



    $$
    g(x)=sum_{n=0}^{infty} frac{(-1)^{n+1}x^{3n}}{(3n+2)!}=frac{e^{-x}left(2e^{3x/2}sin(frac{1}{6}(pi-3sqrt3 x))-1right)}{3x^2},
    $$



    and note that in order to change the properties of the sine wave on the RHS we need to alter the factorial on the LHS. Let $k$ now denote a positive integer $geq0$ and examine what happens when we change $(3n+2)!$ to some $(3n+k)!$. In particular, we examine the three cases $kequiv0 (textrm{mod} 3)$, $kequiv1 (textrm{mod} 3)$ and $kequiv2 (textrm{mod} 3)$.





    Case 1. $k=3j$, $jinmathbb{Z}$.



    Let's examine $k=0$ first and generalize later.



    $$
    g'(x)=sum_{n=0}^{infty} frac{(-1)^{n+1}x^{3n}}{(3n)!}=-frac13 e^{-x} left(2e^{3x/2}cosleft(frac{sqrt{3} x}{2}right)+1right),
    $$



    and by similar methods we get $zeta(3)cong frac87left(frac{pi^{3}}{6(3)^{3/2}}right)=1.136602043ldots$ which is not a very good approximation. Having $j>0$ yields similar results that include the cosine function, but with some leading $exp$ terms. For instance, we have for $j=3$:



    $$
    sum_{n=0}^{infty} frac{(-1)^{n+1}x^{3n}}{(3n+9)!}=frac{e^{-x}left(-e^{x}x^6+120e^xx^3-720e^x+480e^{3x/2}cos(frac{sqrt{3} x}{2})+240right)}{720x^9}
    ,$$



    and we can see that when $xtoinfty$ we are making an even bigger error in our approximation by assuming that its roots are those that satisfy the equation



    $$
    480cosleft(frac{sqrt{3} x}{2}right)=0.
    $$



    In fact, as $j$ increases so does our error.





    Case 2. $k=3j+1$, $jinmathbb{Z}$.



    In this case we get similar expressions as your original sine wave, but with some differences in sign (pun not intended). For $j=0$ we have



    $$
    g'(x)=sum_{n=0}^{infty} frac{(-1)^{n+1}x^{3n}}{(3n+1)!}=-frac{e^{-x}left(2e^{3x/2}sin(frac{1}{6}(pi+3sqrt3 x))-1right)}{3x^2},
    $$



    which has the same roots as your original function $g(x)$ and the same error in approximating $zeta(3)$. As in Case 1, increasing the value of $j$ leads to even more leading $exp$ terms and thus increases the error in our approximation.





    Case 3. $k=3j+2$, $jinmathbb{Z}$.



    The case $j=0$ corresponds to your original function $g(x)$. As with cases 1 and 2, increasing $j$ increases the amount of leading $exp$ terms and thus the error in the approximation due to taking the roots of the asymptotic value when $xtoinfty$.





    It remains to be seen whether it is possible to sensibly generalize the factorial to $(in+j)!$, $i,jinmathbb{Z}, i>0$ in order to get other approximations for other odd zeta values, perhaps several of these cases might yield good approximations. Euler himself tried to generalize his proof of the Basel problem to compute values for $sum_n n^{-(2k+1)}$, $k>0$ but found no way to achieve this. Since then, considerable brain power has been expended to do just that, without success. I believe that is a good indicator that this path might not lead to a nice closed-form expression for $zeta(3)$ in terms of $pi^3$.





    EDIT 11.12.2018



    Taking the function



    $$
    sum_{n=0}^{infty} frac{(-1)^{n+1}x^{5n}}{(3n+2)!}=frac{e^{-x^{5/3}}left(2e^{3x^{5/3}/2}sin(frac{1}{6}(pi-3sqrt3 x^{5/3}))-1right)}{3x^{10/3}}
    $$



    yields the same approximation as $g(x)$, i.e., with $x^{3n}$ instead of $x^{5n}$ in the sum. This provides further indication that this approach is unlinkely to yield any better results on $zeta(3)$.






    share|cite|improve this answer











    $endgroup$


















      2












      $begingroup$

      First of all, thank you for the highly interesting question, it provides an excellent exercise!





      Unfortunately it is not possible to improve the approximation by your method alone. Of course, the error in your approximation is due to taking the roots of $2sin(frac{pi}{6}-frac{sqrt{3}}{2}x)=e^{-frac{3x}{2}}$ when $xtoinfty$ and thus using the asymptotic $2sin(frac{pi}{6}-frac{sqrt{3}}{2}x)=0$ instead. In order to alter this error, we examine your function $g(x)$:



      $$
      g(x)=sum_{n=0}^{infty} frac{(-1)^{n+1}x^{3n}}{(3n+2)!}=frac{e^{-x}left(2e^{3x/2}sin(frac{1}{6}(pi-3sqrt3 x))-1right)}{3x^2},
      $$



      and note that in order to change the properties of the sine wave on the RHS we need to alter the factorial on the LHS. Let $k$ now denote a positive integer $geq0$ and examine what happens when we change $(3n+2)!$ to some $(3n+k)!$. In particular, we examine the three cases $kequiv0 (textrm{mod} 3)$, $kequiv1 (textrm{mod} 3)$ and $kequiv2 (textrm{mod} 3)$.





      Case 1. $k=3j$, $jinmathbb{Z}$.



      Let's examine $k=0$ first and generalize later.



      $$
      g'(x)=sum_{n=0}^{infty} frac{(-1)^{n+1}x^{3n}}{(3n)!}=-frac13 e^{-x} left(2e^{3x/2}cosleft(frac{sqrt{3} x}{2}right)+1right),
      $$



      and by similar methods we get $zeta(3)cong frac87left(frac{pi^{3}}{6(3)^{3/2}}right)=1.136602043ldots$ which is not a very good approximation. Having $j>0$ yields similar results that include the cosine function, but with some leading $exp$ terms. For instance, we have for $j=3$:



      $$
      sum_{n=0}^{infty} frac{(-1)^{n+1}x^{3n}}{(3n+9)!}=frac{e^{-x}left(-e^{x}x^6+120e^xx^3-720e^x+480e^{3x/2}cos(frac{sqrt{3} x}{2})+240right)}{720x^9}
      ,$$



      and we can see that when $xtoinfty$ we are making an even bigger error in our approximation by assuming that its roots are those that satisfy the equation



      $$
      480cosleft(frac{sqrt{3} x}{2}right)=0.
      $$



      In fact, as $j$ increases so does our error.





      Case 2. $k=3j+1$, $jinmathbb{Z}$.



      In this case we get similar expressions as your original sine wave, but with some differences in sign (pun not intended). For $j=0$ we have



      $$
      g'(x)=sum_{n=0}^{infty} frac{(-1)^{n+1}x^{3n}}{(3n+1)!}=-frac{e^{-x}left(2e^{3x/2}sin(frac{1}{6}(pi+3sqrt3 x))-1right)}{3x^2},
      $$



      which has the same roots as your original function $g(x)$ and the same error in approximating $zeta(3)$. As in Case 1, increasing the value of $j$ leads to even more leading $exp$ terms and thus increases the error in our approximation.





      Case 3. $k=3j+2$, $jinmathbb{Z}$.



      The case $j=0$ corresponds to your original function $g(x)$. As with cases 1 and 2, increasing $j$ increases the amount of leading $exp$ terms and thus the error in the approximation due to taking the roots of the asymptotic value when $xtoinfty$.





      It remains to be seen whether it is possible to sensibly generalize the factorial to $(in+j)!$, $i,jinmathbb{Z}, i>0$ in order to get other approximations for other odd zeta values, perhaps several of these cases might yield good approximations. Euler himself tried to generalize his proof of the Basel problem to compute values for $sum_n n^{-(2k+1)}$, $k>0$ but found no way to achieve this. Since then, considerable brain power has been expended to do just that, without success. I believe that is a good indicator that this path might not lead to a nice closed-form expression for $zeta(3)$ in terms of $pi^3$.





      EDIT 11.12.2018



      Taking the function



      $$
      sum_{n=0}^{infty} frac{(-1)^{n+1}x^{5n}}{(3n+2)!}=frac{e^{-x^{5/3}}left(2e^{3x^{5/3}/2}sin(frac{1}{6}(pi-3sqrt3 x^{5/3}))-1right)}{3x^{10/3}}
      $$



      yields the same approximation as $g(x)$, i.e., with $x^{3n}$ instead of $x^{5n}$ in the sum. This provides further indication that this approach is unlinkely to yield any better results on $zeta(3)$.






      share|cite|improve this answer











      $endgroup$
















        2












        2








        2





        $begingroup$

        First of all, thank you for the highly interesting question, it provides an excellent exercise!





        Unfortunately it is not possible to improve the approximation by your method alone. Of course, the error in your approximation is due to taking the roots of $2sin(frac{pi}{6}-frac{sqrt{3}}{2}x)=e^{-frac{3x}{2}}$ when $xtoinfty$ and thus using the asymptotic $2sin(frac{pi}{6}-frac{sqrt{3}}{2}x)=0$ instead. In order to alter this error, we examine your function $g(x)$:



        $$
        g(x)=sum_{n=0}^{infty} frac{(-1)^{n+1}x^{3n}}{(3n+2)!}=frac{e^{-x}left(2e^{3x/2}sin(frac{1}{6}(pi-3sqrt3 x))-1right)}{3x^2},
        $$



        and note that in order to change the properties of the sine wave on the RHS we need to alter the factorial on the LHS. Let $k$ now denote a positive integer $geq0$ and examine what happens when we change $(3n+2)!$ to some $(3n+k)!$. In particular, we examine the three cases $kequiv0 (textrm{mod} 3)$, $kequiv1 (textrm{mod} 3)$ and $kequiv2 (textrm{mod} 3)$.





        Case 1. $k=3j$, $jinmathbb{Z}$.



        Let's examine $k=0$ first and generalize later.



        $$
        g'(x)=sum_{n=0}^{infty} frac{(-1)^{n+1}x^{3n}}{(3n)!}=-frac13 e^{-x} left(2e^{3x/2}cosleft(frac{sqrt{3} x}{2}right)+1right),
        $$



        and by similar methods we get $zeta(3)cong frac87left(frac{pi^{3}}{6(3)^{3/2}}right)=1.136602043ldots$ which is not a very good approximation. Having $j>0$ yields similar results that include the cosine function, but with some leading $exp$ terms. For instance, we have for $j=3$:



        $$
        sum_{n=0}^{infty} frac{(-1)^{n+1}x^{3n}}{(3n+9)!}=frac{e^{-x}left(-e^{x}x^6+120e^xx^3-720e^x+480e^{3x/2}cos(frac{sqrt{3} x}{2})+240right)}{720x^9}
        ,$$



        and we can see that when $xtoinfty$ we are making an even bigger error in our approximation by assuming that its roots are those that satisfy the equation



        $$
        480cosleft(frac{sqrt{3} x}{2}right)=0.
        $$



        In fact, as $j$ increases so does our error.





        Case 2. $k=3j+1$, $jinmathbb{Z}$.



        In this case we get similar expressions as your original sine wave, but with some differences in sign (pun not intended). For $j=0$ we have



        $$
        g'(x)=sum_{n=0}^{infty} frac{(-1)^{n+1}x^{3n}}{(3n+1)!}=-frac{e^{-x}left(2e^{3x/2}sin(frac{1}{6}(pi+3sqrt3 x))-1right)}{3x^2},
        $$



        which has the same roots as your original function $g(x)$ and the same error in approximating $zeta(3)$. As in Case 1, increasing the value of $j$ leads to even more leading $exp$ terms and thus increases the error in our approximation.





        Case 3. $k=3j+2$, $jinmathbb{Z}$.



        The case $j=0$ corresponds to your original function $g(x)$. As with cases 1 and 2, increasing $j$ increases the amount of leading $exp$ terms and thus the error in the approximation due to taking the roots of the asymptotic value when $xtoinfty$.





        It remains to be seen whether it is possible to sensibly generalize the factorial to $(in+j)!$, $i,jinmathbb{Z}, i>0$ in order to get other approximations for other odd zeta values, perhaps several of these cases might yield good approximations. Euler himself tried to generalize his proof of the Basel problem to compute values for $sum_n n^{-(2k+1)}$, $k>0$ but found no way to achieve this. Since then, considerable brain power has been expended to do just that, without success. I believe that is a good indicator that this path might not lead to a nice closed-form expression for $zeta(3)$ in terms of $pi^3$.





        EDIT 11.12.2018



        Taking the function



        $$
        sum_{n=0}^{infty} frac{(-1)^{n+1}x^{5n}}{(3n+2)!}=frac{e^{-x^{5/3}}left(2e^{3x^{5/3}/2}sin(frac{1}{6}(pi-3sqrt3 x^{5/3}))-1right)}{3x^{10/3}}
        $$



        yields the same approximation as $g(x)$, i.e., with $x^{3n}$ instead of $x^{5n}$ in the sum. This provides further indication that this approach is unlinkely to yield any better results on $zeta(3)$.






        share|cite|improve this answer











        $endgroup$



        First of all, thank you for the highly interesting question, it provides an excellent exercise!





        Unfortunately it is not possible to improve the approximation by your method alone. Of course, the error in your approximation is due to taking the roots of $2sin(frac{pi}{6}-frac{sqrt{3}}{2}x)=e^{-frac{3x}{2}}$ when $xtoinfty$ and thus using the asymptotic $2sin(frac{pi}{6}-frac{sqrt{3}}{2}x)=0$ instead. In order to alter this error, we examine your function $g(x)$:



        $$
        g(x)=sum_{n=0}^{infty} frac{(-1)^{n+1}x^{3n}}{(3n+2)!}=frac{e^{-x}left(2e^{3x/2}sin(frac{1}{6}(pi-3sqrt3 x))-1right)}{3x^2},
        $$



        and note that in order to change the properties of the sine wave on the RHS we need to alter the factorial on the LHS. Let $k$ now denote a positive integer $geq0$ and examine what happens when we change $(3n+2)!$ to some $(3n+k)!$. In particular, we examine the three cases $kequiv0 (textrm{mod} 3)$, $kequiv1 (textrm{mod} 3)$ and $kequiv2 (textrm{mod} 3)$.





        Case 1. $k=3j$, $jinmathbb{Z}$.



        Let's examine $k=0$ first and generalize later.



        $$
        g'(x)=sum_{n=0}^{infty} frac{(-1)^{n+1}x^{3n}}{(3n)!}=-frac13 e^{-x} left(2e^{3x/2}cosleft(frac{sqrt{3} x}{2}right)+1right),
        $$



        and by similar methods we get $zeta(3)cong frac87left(frac{pi^{3}}{6(3)^{3/2}}right)=1.136602043ldots$ which is not a very good approximation. Having $j>0$ yields similar results that include the cosine function, but with some leading $exp$ terms. For instance, we have for $j=3$:



        $$
        sum_{n=0}^{infty} frac{(-1)^{n+1}x^{3n}}{(3n+9)!}=frac{e^{-x}left(-e^{x}x^6+120e^xx^3-720e^x+480e^{3x/2}cos(frac{sqrt{3} x}{2})+240right)}{720x^9}
        ,$$



        and we can see that when $xtoinfty$ we are making an even bigger error in our approximation by assuming that its roots are those that satisfy the equation



        $$
        480cosleft(frac{sqrt{3} x}{2}right)=0.
        $$



        In fact, as $j$ increases so does our error.





        Case 2. $k=3j+1$, $jinmathbb{Z}$.



        In this case we get similar expressions as your original sine wave, but with some differences in sign (pun not intended). For $j=0$ we have



        $$
        g'(x)=sum_{n=0}^{infty} frac{(-1)^{n+1}x^{3n}}{(3n+1)!}=-frac{e^{-x}left(2e^{3x/2}sin(frac{1}{6}(pi+3sqrt3 x))-1right)}{3x^2},
        $$



        which has the same roots as your original function $g(x)$ and the same error in approximating $zeta(3)$. As in Case 1, increasing the value of $j$ leads to even more leading $exp$ terms and thus increases the error in our approximation.





        Case 3. $k=3j+2$, $jinmathbb{Z}$.



        The case $j=0$ corresponds to your original function $g(x)$. As with cases 1 and 2, increasing $j$ increases the amount of leading $exp$ terms and thus the error in the approximation due to taking the roots of the asymptotic value when $xtoinfty$.





        It remains to be seen whether it is possible to sensibly generalize the factorial to $(in+j)!$, $i,jinmathbb{Z}, i>0$ in order to get other approximations for other odd zeta values, perhaps several of these cases might yield good approximations. Euler himself tried to generalize his proof of the Basel problem to compute values for $sum_n n^{-(2k+1)}$, $k>0$ but found no way to achieve this. Since then, considerable brain power has been expended to do just that, without success. I believe that is a good indicator that this path might not lead to a nice closed-form expression for $zeta(3)$ in terms of $pi^3$.





        EDIT 11.12.2018



        Taking the function



        $$
        sum_{n=0}^{infty} frac{(-1)^{n+1}x^{5n}}{(3n+2)!}=frac{e^{-x^{5/3}}left(2e^{3x^{5/3}/2}sin(frac{1}{6}(pi-3sqrt3 x^{5/3}))-1right)}{3x^{10/3}}
        $$



        yields the same approximation as $g(x)$, i.e., with $x^{3n}$ instead of $x^{5n}$ in the sum. This provides further indication that this approach is unlinkely to yield any better results on $zeta(3)$.







        share|cite|improve this answer














        share|cite|improve this answer



        share|cite|improve this answer








        edited Dec 11 '18 at 13:20

























        answered Dec 7 '18 at 15:40









        KlangenKlangen

        1,72811334




        1,72811334






























            draft saved

            draft discarded




















































            Thanks for contributing an answer to Mathematics Stack Exchange!


            • Please be sure to answer the question. Provide details and share your research!

            But avoid



            • Asking for help, clarification, or responding to other answers.

            • Making statements based on opinion; back them up with references or personal experience.


            Use MathJax to format equations. MathJax reference.


            To learn more, see our tips on writing great answers.




            draft saved


            draft discarded














            StackExchange.ready(
            function () {
            StackExchange.openid.initPostLogin('.new-post-login', 'https%3a%2f%2fmath.stackexchange.com%2fquestions%2f1459709%2fcan-this-approximate-closed-form-of-aperys-constant-zeta3-be-improved%23new-answer', 'question_page');
            }
            );

            Post as a guest















            Required, but never shown





















































            Required, but never shown














            Required, but never shown












            Required, but never shown







            Required, but never shown

































            Required, but never shown














            Required, but never shown












            Required, but never shown







            Required, but never shown







            Popular posts from this blog

            Le Mesnil-Réaume

            Ida-Boy-Ed-Garten

            web3.py web3.isConnected() returns false always